Вы находитесь на странице: 1из 45

Criminal Week 1 Principles of punishment 8/16 and 8/18 Proportionality- 3 factor test 1.

The gravity of the offense and the harshness of the penalty for that offense. 2. The sentences imposed on other criminals in the same jurisdiction. 3. The sentences imposed for commission of the same crime in other jurisdictions. Deterrence- Rehabilitation, Isolation, Education, Retribution. Continued in detail on pg. 5 When determining the particular sentence to be imposed, the Court shall consider1. The nature and circumstances of the offense and the history and characteristics of the defendant. 2. The need for the sentence to be imposed- to reflect the seriousness of the offense, to promote respect for the law, and to provide just punishment for the offense. To afford adequate deterrence to criminal conduct. To protect the public from further crimes of the defendant and to provide D with needed educational or vocational training, medical care, or other correctional treatment in the most effective manner. 3. The kinds of sentences available. 4. The kinds of sentence and sentencing range established for the applicable category of offense committed by the applicable category of defendant as set forth in the guidelines. Pg. 21

The Queen v. Dudley and Stephens (1884-85) stranded at sea cannibals Rule: a crime incurs a formal and solemn pronouncement of the moral condemnation of the community

State v. Jensen (Idaho 2002) insulin shot murderer Rule: Burden is on appellate to prove abuse of discretion.

US v. Crawford (9th circuit 2008) drug dealer sentencing appeal Rule: In Blakely v. Washington the 6th amendment (right to jury trial) prohibited judges from enhancing criminal sentences based on facts other than those decided by the jury or admitted by the defendant. D asked to recall the sentence. In US v Booker sentencing guidelines were advisory, not mandatory, appellate courts should review sentences for reasonableness. Kimbrough v. US- sentences regarding crack cocaine are only advisory, Ds sentence was OK. In Rita it was said that the guidelines are the starting point and initial benchmark, they are to be kept in mind throughout the sentencing process.

Ewing v. California (U.S. 11 2003) three strikes and youre out Rule: Requires that a when D is convicted of a felony, and has previously been convicted of a serious or violent felony, then the sentence imposed under these 3 strikes law shall include a 5 year enhancement. If D has 2 prior serious or violent felonies, min term of imprisonment is 25 years. We must place the scales not only on his current felony, but also his history of felony recidivism. States have valid interest in deterring and segregating habitual criminals Ropper v. Simmons (US 2005) death penalty for minors, cruel and unusual punishment Rule: Furman v. Ga. - 8th amendment restricts excessive bail/fines or cruel and unusual punishment. It is applicable to the state through the 14th amendment. Atkins v. Va. 8th and 14th do not allow executing a mentally retarded person. Trop v. Dulles- the court noted that the progress of a maturing society and its standards are to be taken into account during the punishment phase. Thompson v. Ok. - plurality of the court said that decency standards do not allow the execution of someone under 16 at the time the crime was committed. Stanford v. Ky. the court ruled 5/4 that the 8th and 14th did not proscribe executing 15-18 year olds. There was no national consensus sufficient to label a particular punishment cruel and unusual. Penry v. Lynaugh- 8th did not mandate a categorical exemption from the death penalty for the mentally retarded. But since Penry, standards evolved and it is cruel and unusual to execute a retarded person. No national legislative consistency proscribing juvenile execution since Stanford. But now, considering recent legislation, the court holds that offenders under 18 cannot be executed required by the 8th amendment.

Week 2 8/23 and 8/25 Criminal Statutes Actus Reus The Basics of a Criminal Prosecution 1. The Investigation- Conducted by Police- gather evidence arrest Def. recommend charges to Prosecutor 2. The Charge- Determined by prosecutor 3. Pre-Trial Motions: Examples- exclusion of evidence, request to dismiss charges for insufficient evidence or failure to charge crime. Courts conduct preliminary hearing to review whether there is probable cause. 4. Trial- Constitution entitles Def. to jury trial if he can be incarcerated for more than 6 months for offense. For most minor cases, courts decided case ** (6th Amendment!!)** 5. Plea Bargain- Through counsel, can agree to a plea bargain; Def will plead guilty to certain charges in exchange for lower sentence. Requires Def to waive his right to trial and proceed immediately to sentencing. 6. Jury Instruction- Contain description of the elements of the crime, must find beyond a reasonable doubt. 7. Motion for Judgment of Acquittal- Similar to Summary Judgment, Def can only seek to have case dismissed for insufficient evidence. 8. Double Jeopardy- Def. may not be put twice in jeopardy for same offense. If person is found not guilty, government may not pursue that in charge in a second proceeding. One exception, Dual Sovereignty Doctrine, permits different state or federal government has jurisdiction over offense. 9. The Basis for Appeal- Insufficient evidence, improper jury instruction, evidentiary challenges (when evidence improperly admitted), Constitutional challenge. I. Criminal Statutes and Interpretations of Statutes a. Divided into 2 categories: i. Felonies- more than 1 year 1 day ii. Misdemeanors- Less than 1 year 1 day b. Malum in se- conduct wrongful in itself (carjacking)
2

c. Malum prohibitum- wrongful through legislative prohibition or statute (rape, murder, larceny) d. The Model Penal Code (MPC) i. One of the most important statutory developments in crim law. ii. Drafted by American Law Institute (ALI), iii. Widely influential in states drafting their own penal statutes e. Interpreting Statutory Language- 2 ways i. Archeological- meaning of statute is set in stone on date of enactment ii. Nautical- views statute as an on-going process f. Defining a Crime- first question when analyzing a case i. WHAT DOES THE STATUTE SAY People v. Lopez (Cal 2003) carjacking Rule: But taking was given a higher meaning rather than carrying away because carjacking was elevated from a second degree robbery to a felony due to section 215s creation because of the increase in thrill seeking carjackers. Carjacking means temporarily or permanently depriving someone of their car. Robbery requires permanent intent. Taking was expanded from possessor to possessor or any passenger. Robbery is any property, carjacking is only cars. A person can be charged with both crimes simultaneously. But no one can be punished for the same act by both statutes. If a term in the common law has not been defined by statute it is assumed the meaning was intended. Movement of the car is immaterial in carjacking if there was serious potential harm to the victim. Movement is sufficient but not necessary (as in a robbery) to constitute abduction. Movement is necessary to show an observable manifestation of the wrongdoer in the robbery; a stricter standard than a carjacking.

State v. Donaldson (Iowa 2003) attempted/successful carjacking Rule: He asportated the vehicle when he broke in and controlled the electrical equipment. To prove theft the state successfully proved he took the property of another, secured dominion over it, carried the property away. Attempt to move is a misdemeanor while the completion is larceny- a felony. Control is defined as using the object in a manner beyond his authority. Question- did D exercise wrongful dominion or unauthorized control of the van? D said he did not have the ability to readily move the van. But movement is not necessary to maintain possession of the car. When he manipulated the electrical system he exerted complete control over the van. Denial of acquittal affirmed.

Small v. United States Rule: Unlawful gun possession statute- unlawful for any person who has been convicted in any court of a crime punishable by imprisonment for a term exceeding one year to possess any firearm. Even though the word any demands broad interpretation, we must look beyond that word itself.

Columbus v. Kim (Ohio 2008) barking dog Rule: Court of appeals said that a person of ordinary intelligence would know what conduct the ordinance prohibited. Coates v. Cincinnati- no standard of conduct is specified at all, not meaning that a person must conform to imprecise standards. D must prove beyond a reasonable doubt that the statute was so unclear that
3

she could not reasonably understand that it prohibited her actions. The statute offered no standard of barking because everyones sensitivities are different. The court held that a reasonable person would know what conduct is normal and what is prohibited under the statute. D did not prove that the statute provided no standards. Judgment affirmed. A court considering a vagueness issue asks whether a person of ordinary intelligence can understand the terms used by the legislature, but not whether the particular person in fact knew what the statute means.

I.

Actus Reus a. Elements of a crime i. The act (actus Reus) ii. The mental state (mens rea) iii. Causation iv. Attendant circumstances. b. Elements of Burglary i. Breaking and entering (actus reus) ii. The dwelling house (attendant circumstances) iii. Of Another (attendant circumstances) iv. At nighttime (attendant circumstances) v. With the intent to commit a felony therein (intent) c. Wrongful act i. A person is not guilty of an offense unless his liability is based on conduct which includes a voluntary act or the omission to perform an act of which he is physically capable d. Voluntary act- the following are NOT voluntary acts under MPC: i. Reflex or convulsion, bodily movement during unconsciousness or sleep, conduct during hypnosis, bodily movement that otherwise is not a product of the effort or determination of the actor, either conscious or habitual

Martin v. State (Ala. App. 1944) drunk in public Rule: There was no evidence/proof for his arrest because he was brought involuntarily to that place.

Cox v. Director of Revenue (Mo. 2003) drunk and sleeping behind the wheel Rule: once the key is in the ignition, engine running, P is in drivers seat, the cop had probable cause that P was operating the vehicle. D properly suspended Ps license.

West v. Commonwealth (Ky. App. 1996) reckless homicide of disabled sister Rule: People v. Beardsley- omission of a duty or act that results in someones death makes the other liable for manslaughter. The duty neglected must be a legal duty, not a moral one. Duty imposed by law or contract, omission to perform must be the immediate and direct cause of death. Jury needed to find P, beyond a reasonable doubt, that he was the caretaker of the victim. P met 4th criteria of the Jones test- P voluntarily assumed care of another and so secluded the helpless person as to prevent others from rendering aid. Judgment affirmed, appeal denied.
4

State v. Winsor (Mo. App. 2003) drugs in prison Rule: D willfully controlled the substance so it constituted a requisite voluntary act. The statute does not require him to be voluntarily in jail. He misconstrued and misapplied the voluntary act aspect of the law. Appeal denied.

People v. Ross (Cal. App. 2008) gun in prison Rule: Legislative intent is to deter people from bringing guns into prisons regardless of if they are under arrest. If D would win then convicts could bring weapons into jail without penalty.

Watson v. State (Fla. App. 2004) Cocaine in the house, whos is it? Rule: Brown v. State- in order to prove constructive possession the state needed to prove that P had control or dominion over the room. A visitor without evidence for control of the drugs could not be convicted. Jury could have found that P was more than a visitor but did not.

Robinson v. Ca (US 2002) To be addicted or not to be, that is the question? Rule: All the state had to do was prove P did or was addicted to drugs in Los Angeles County. The jury was allowed to convict even if they did not believe P did drugs in L.A. all they had to find was the status or chronic condition of being addicted to drugs. Law in California made it illegal to be addicted to drugs, is that constitutional?

Week 3 8/30 and 9/1 Mens Rea Bank robbery and incidental crimes- whoever, by force or violence, or by intimidation, takes, or attempts to take, from the person or presence of another, or obtains or attempts to obtain by extortion any property or money or any other thing of value belonging to, or in the care, custody, control, management, or possession of, any bank, credit union, or any savings and loan association shall be fined under this title or imprisoned not more than twenty years, or both. Whoever takes and carries away, with intent to steal or purloin, any property or money or any other thing of value exceeding $1000 belonging to, or in the care, custody, control, management, or possession of any bank, credit union, or any savings and loan association, shall be fined under this title or imprisoned not more than ten years, or both. MPC model statute rules pgs 107-110- culpability; purposely, knowingly, recklessly, and negligently. MPC statute rules pgs 144-145- transferred intent. MPC statute rules pgs 151-152- culpability requirements, prosecutions burden, and strict liability elements. I. Mens Rea a. Model Penal Code- General Definitions i. Act or Action- bodily movement whether voluntary or involuntary
5

ii. Omission- failure to act iii. Conduct- action or omission and is accompanying state of mind, or, where relevant, a series of acts or omissions iv. Actor- person guilty of omission b. General Requirements of Culpability i. Minimum requirements- a person is not guilty of an offense UNLESS he acted purposely, knowingly, recklessly or negligently, as the law may require, with respect to each material element of offense ii. Kinds of Culpability 1. Purposely- involves the nature of his conduct, conscious object to engage in conduct or obtain such a result, involves attendant circumstances, and he is aware of such circumstances 2. Knowingly- nature of conduct, AWARE that his conduct is of nature or that such attendant circumstances exist, aware and practically certain will produce such a result. 3. Recklessly- consciously disregards a substantial and unjustifiable risk that the material element exists or will result from his conduct, involves a gross deviation from the standard of conduct that a law-abiding person would observe in the actors situation. 4. Negligently- when he should be aware of a substantial and unjustifiable risk that the material element exists or will result from his conduct. The risk must be of such a nature and degree that the actors failure to perceive it, considering the nature and purpose of his conduct and circumstances known to him. c. MPC Interpreting Mens Rea i. Level of culpability required unless otherwise provided- 2.02(3) ii. Prescribed culpability apples to all material elements- 2.02 (4) iii. Substitutes for negligence, recklessness, and knowledge- 2.02 (5) iv. Requirement for purpose satisfied if purpose is conditional- 2.02 (6) v. Knowledge satisfied by Knowledge of High Probabtility- 2.02 (7) vi. Wilfullness satisfied by acting knowingly- 2.02(8) vii. Ignorance of the law is no excuse- 2.02 (9) viii. Where the degree of offense depends on level of culpability, look to element proven at the lowest level court- 2.02 (10) II. General intent a. Government must prove that D engaged consciously in the conduct b. But need not prove that D intended any particular result or even that he was fully aware he was committing a crime MPC states of mind, with respect to material elements- Purposely- Trombley, Knowingly- Youts, RecklesslyHall, Negligently- Larson. Purposely- Nature of conduct- conscious object to engage in conduct or obtain such result. Attendant circumstances- aware of them or believes or hopes they exist. Knowingly- Nature of conduct- aware that conduct is of that nature and that such circumstances exists. Result of conduct- aware that it is practically certain his conduct will produce the result. Recklessly- Consciously disregards substantial unjustifiable risk. Risk is such that its disregard is a gross deviation from standard of conduct a law-abiding citizen would observe.
6

Negligently- should be aware of substantial and unjustifiable risk. Risk such that failure to perceive the risk is a gross deviation from standard of care a reasonable person would observe. MPC Mens Rea- Level of culpability required unless otherwise provided. Prescribed culpability applies to all material elements. Substitutes for negligence, recklessness, and knowledge. Purpose satisfied even if purpose is conditional. Knowledge satisfied by high probability. Willfulness satisfied by acting knowingly. Ignorance of the law is no excuse. Where degree of offense depends on level of culpability, look to the element proven at the lowest level. Mistake of Fact Mens Rea Specific Intent General Intent Strict Liability Application Any mistake is a defense, even unreasonable mistakes Reasonable mistakes are a defense No defense due to any mistake

Mistake of fact or law is a valid defense if the mistake negatives the intent element of the offense. Cant claim mistake if what you thought you were doing was a crime

Carter v. US (US 2000) bank robber says he did not mean to permanently take the money Rule: there needs to be intent to steal to be charged under the statutes part B, no force component. Part A has 20 years in jail, part B only has 10. Section A contains no intent to steal or purloin, section B requires it. Section A only requires general intent. A court only has to read into a statute only that mens rea which is necessary to separate wrongful conduct from otherwise innocent conduct. There needs to be specific intentintent to steal or purloin rather than just find and take peacefully. General intent- proof of knowledge with respect to the actus reus of the crime. There does not need to be specific intent for the intent to steal or purloin because the intent for a forceful taking, even in good faith, falls outside the realm of otherwise innocent.

Vermont v. Trombley (Vt. 2002) intent in aggravated assault Rule: MPC, 4 kinds of culpability- purposely, knowingly, recklessly, and negligently; replaced general and specific intent. Vermont changed its law to include attempts to cause SBI. D bears the burden of producing evidence for self defense claim against victim. But self defense means that he acted with purpose to inflict SBI. Did D act purposely and knowingly when charged with only purposely inflicting serious bodily injury (SBI)?

US v. Youts (10th Cir. 2000) drunk steals a train and wrecks it Rule: The court would reverse only if no rational trier of fact could have found the essential elements of the crime beyond a reasonable doubt. The federal train wreck statute punished anyone who willfully derails, disables, or wrecks any train, engine, motor unit, or car used, operated, or employed in interstate or foreign commerce by any railroad.

People v. Hall (Colo. 2000) ski patrol kills skier Rule: for D to be reckless his conduct must have been at least more likely than not that death would result. For conduct to be reckless the actor must have consciously disregarded a substantial and unjustifiable risk that death could result from his actions. Prosecution must show that D consciously disregarded a substantial and unjustifiable risk that he would cause the death of another. The risk must be of such a nature that its disregard constitutes a gross deviation from the standard of care that a reasonable person would exercise. Prosecution only needs to establish sufficient evidence.

State v. Larson (Mont. 2004) drunk driver kills passenger Rule: negligent conduct- act is done with a conscious disregard of the risk that death of a human being will occur or that the circumstance exists or when the person disregards a risk of causing the death of another human being which the person should be aware that the result will occur or that the circumstance exists. Risk must be of a nature and degree that to disregard it involves a gross deviation from the standard of conduct that a reasonable person would observe in the actors situation. Gross deviation is deviation that is considerably greater than lack of ordinary care. Mental state is not an issue in negligent homicide cases (State v. Gould). Negligent homicide does not require purpose or knowledge.

Holloway v. US (US 1999) proving intent to kill in carjacking Rule: A D may intend to engage in certain conduct only if a certain event occurs. Intent in carjacking can be conditional or unconditional. People v. Connors- specific intent to kill could be found even though that intent was coupled with a condition that D would not fire if the victim complied with his demand. D may not negate a proscribed intent by requiring the victim to comply with a condition D has no right to impose. Government must prove beyond a reasonable doubt that D would have at least attempted to seriously harm or kill the driver if that action had been necessary to complete the taking of the car. Does the government need to prove D had an unconditional intent to kill or harm if necessary to effect a carjacking?

Rice v. State (Md. Special App. 2000) driving on suspended license Rule: willful blindness and deliberate ignorance was enough to convict. Criminal intent required for driving while suspended is knowledge rather than intent but intentionally blinding yourself to the truth or probable fact(s) is enough for intent.

State v. Fennell (SC 2000) crazy guy gets life and then some Rule: mental state- intent is limitless. Killing the intended victim or not does not matter. If someone plans on murdering someone they are responsible for knowing that there may be unintended victims.

People v. Nasir (Mich. App. 2003) counterfeit stamps mistake Rule: statute created a strict liability offense. Prosecution had to prove beyond a reasonable doubt that D possessed or used a counterfeit stamp at the store without authorization. Even if the legislature did not expressly include language indicating that fault is a necessary element of a crime, the court focuses on whether the legislature intended to require some fault as a predicate to finding guilt. Knowledge is an element of the offense. Legislature did not intend the offense to contain a specific intent element, and D did not need to act with knowledge that D does so without the authorization of the Michigan Dept. of Treasury. Did the legislature intend to dispense with a mens rea or fault requirement when creating this offense? It does not include a fault element.

Week 4 9/6 and 9/8 Mens Rea: Mistake. Causation, Concurrence, Merger Mistake of fact- an act committed or an omission made under a mistake of fact which nullifies the requisite intent as an element of the crime charged is a defense to that crime. Thus a person is not guilty of a crime if he commits an act or omits to act under a honest and reasonable belief in the existence of certain facts and circumstances which, if true, should make such act or omission lawful. Naugher Rule- a defendant has a right to an affirmative instruction on his theory of the case when there is competent and substantial evidence to sustain that theory and the offered instruction is sufficient to apprise the trial court of the theory. Under those circumstances, the failure to affirmatively instruct the jury with respect to the Ds theory of the case violates due process and is prejudicial error. General and specific intent- under the common law a Ds claimed mistake for a specific intent crime did not have to be reasonable. In contrast, for a general intent crime, the Ds claimed mistake must be a reasonable one, a mistake that a person of ordinary care and prudence would make in similar circumstances. Ds burden is higher for general intent crimes because the jury must first determine whether the D actually had the mistaken belief, and then that the mistake is one a reasonable person would make. MPC Mistake of Fact and Law pgs 167-168- ignorance or mistake, negligence. If statute is silent- prosecution must prove at least negligence. 3 exceptions to mistake of law mistakes that negate mens rea- authorized reliance doctrine, due process limitations, and I. MPC- mistake of fact or law is valid defense if the mistake negates the intent element of the offense. Cant claim mistake if what you thought you were doing was a crime. Causation a. Elements for Cause-in-Fact i. MPC- Conduct is the cause of a result when: 1. It is an antecedent but for which the result in question would not have occurred; AND 2. The relationship between the conduct and result satisfies any additional causal requirements 3. When purposely or knowingly causing a particular result, element not established unless: a. Actual result differs from that designed or contemplated; or b. Actual result involves same kind of injury or harm 4. When reckless or negligently causing particular result, element is not established unless;
9

a. Actual result differs from probable result only in respect that different person or property is injured of affects and harm would have been more serious; or b. Actual result involves same kind of injury and not too remote or accidental bearing on actors liability. b. Elements for Proximate Cause i. MPC- A judgment of responsibility by the jury in deciding whether the particular result was not too remote or accidental in its occurrence to have a just bearing on liability. c. De Minimis Causes: Cause must be more than a trifle i. To be considered the proximate cause of the victims death, Ds act must have been a substantial factor contributing to the result, rather than insignificant or merely theoretical.

Stagner v. Wyoming (Wyo. 1992) stolen car debacle. Rule: Ps testimony that she was told twice about the trade must be taken as entirely true for purposes of testing the competency of evidence needed to support a theory of defense jury instruction.

Cheek v. US (US 1992) crazy pilot doesnt pay his taxes Rule: Supreme Court disagrees that a claimed good faith belief must be objectively reasonable if it is to be considered as possibly negating the governments evidence purporting to show Ds awareness of the legal duty at issue. D can claim anything he wants to negate willfulness; the government is free to present any evidence to the contrary.

Ratzlaf v. US (US 1994) gambler gets into hot water with the feds. Rule: any transaction over $10k in cash had to be reported to the government. But a cashiers check does not need reporting. But banks report anything over $10k

Causation- for crimes that require a specific result, such as death or financial harm, the government must prove beyond a reasonable doubt that the D caused the particular harm as an element of the offense. 2 components to causationCause-in-fact- requires the government to prove that but for actions of the D the result would not have happened when it happened. The D must be the proximate cause of the result. Proximate cause is a means to determine whether the D, as a matter of fairness, should be held liable for the criminal activity. Proximate cause limits the actual or but for causation to that which is directly related to the act. It is narrower in criminal context than in torts. Involuntary manslaughter- the unintentional killing of a human being without malice, proximately caused byan unlawful act not amounting to a felony nor naturally dangerous to human life, or a culpably negligent act or omission. The state must prove that Ds action was the cause-in-fact (actual case) and the proximate cause (legal cause) of the victims death to satisfy the causation element.
10

Concurrent sufficient cause- either alone would have caused result, both parties are liable. Accelerating an inevitable result- both liable. Obstructed cause- first act might have contributed to the result but 2nd act was sufficient so the 1st act did notnever had a chance. Only second actor is liable.

Causation rules pgs 175-176, 187-188. MPC Causation pgs 182-183.


Concurrence- the elements of the offense must all concur. In most cases one cannot form the intent after the act occurs, and if an act or attendant circumstance is an element of the crime, then the government must prove that it existed at the time of the offense. Coincidental intervening causes- if the intervening cause is merely a coincidence rather than a response, then the question is whether it was foreseeable. The contributed to standard is incorrect, as well as coincidental. Sole substantial cause is the correct standard. Intervening cause- an event that comes between the initial event in a sequence and the end result, thereby altering the natural course of events that might have connected a wrongful act to an injury. State v. Lane (NC App. 1994) punching a drunk is not a good idea Rule: in considering a motion to dismiss the trial court must determine whether substantial evidence of each element of the offense exists. Court must consider the evidence in the light most favorable to the state. Against motion to dismiss let the jury decide. Did the state have sufficient evidence that Ds act of hitting victim was both the actual and legal cause of his death?

Oxendine v. State (Del. 1987) asshole beat his kid to death and sort of got away with it Rule: the state had to show Ds conduct caused the victims death. Causation- antecedent but for which the result in question would not have occurred. If 2nd injury caused victim to die sooner than with only 1st injury; 2nd actor is liable for death. Doctors testimony that something is possible is no evidence at all.

State v. Lamprey (NH 2003) swervies! Rule: causation is an element of 1st degree assault and manslaughter. State needed to prove not only that the prohibited result would not have occurred but for the conduct of D, but also that Ds conduct was the legal (proximate cause) of the prohibited result. Cause must be direct and substantial not merely possible or contributing. Legal causation is defeated by an intervening cause only when the intervening cause amounts to the sole substantial cause of the prohibited result. State v. Seymour- a legal cause is the cause without which the event would not have occurred, and the predominating cause, a substantial factor from which the event follows as a natural, direct and immediate consequence.

11

State v. Pelham (NJ 2003) drunk driver kills innocent guy Rule: causation, 2 parts- a but for test under which Ds conduct is deemed a cause of the event if the event would not have occurred without that conduct and when applicable a culpability assessment- State v. Martin.

Week 5 9/13 and 9/15 Homicide- Murder, Manslaughter Common Law I. Homicide: Murder a. Common Law Definition of Homicide: An unlawful (that is unjustified or inexcusable) killing of a human being b. Common Law Definition of Murder: An unlawful killing done with malice aforethought c. Common Law Definition of Manslaughter: An unlawful killing that is not done with malice aforethought d. Elements of Major Common Law Homicide: a. First Degree Murder i. Willful, deliberate, and premeditated (that is, purposely killing while fully appreciating the significance of your actions and thinking about them beforehand; also known as in cold blood) or ii. Felony murder as specified by statute; or iii. Killing by poison or lying in wait b. Second Degree Murder i. Any intentional (that is, purposeful or knowing) killing other than first degree murder or voluntary manslaughter (for example, a purposeful killing done while unreasonably in the heat of passion); or ii. Felony murder (if not raised to first degree by statute); or iii. Depraved heart (or abandoned and malignant heart) murder (a special kind of extremely reckless killing that exhibits indifference to the value of human life). c. Voluntary Manslaughter i. A killing that would otherwise be murder but that is mitigated to manslaughter because it was done upon being reasonably provoked into a sudden heat of passion without cooling off where a reasonable person would not have cooled off. a. Mental state that would ordinarily qualify as murder b. When the D has been provoked by the victim into the heat of passion ii. Imperfect Self-Defense d. Involuntary Manslaughter i. A reckless killing (provided that it is not of the special type of recklessness that elevates the crime to depraved heart murder); a criminally negligent killing; a killing done with ordinary tort negligence ii. Misdemeanor-manslaughter e. Distinguishing Willful, Deliberate, and Premeditated a. Willfulness: the intent to kill. There need be no appreciable space of time between formation of the intent to kill and the act of killing b. Deliberation: process of determining upon a course of action to kill as a result of thought, including weighing the reasons for and against the action and considering the consequences of the action.
12

c. Premeditation: design, or a determination to kill, distinctly formed in the mind by the time of the killing. Premeditation need not be for a day, and hour or even a minute. It may be as instantaneous as successive thoughts of the mind

Mens Rea Malice- premeditation; some felonies Malice- unreasonable heat; intent to cause serious bodily injury; extreme recklessness; felony No malice but purposely or knowingly; heat with adequate provocation Recklessness Gross negligence Simple negligence MPC Criminal Homicide

Common Law Murder 1st degree Murder 2nd degree

MPC Murder

Murder

Manslaughter- voluntary Manslaughter- involuntary Manslaughter- involuntary Not Homicide

Manslaughter Manslaughter Manslaughter/negligent homicide Negligent homicide

Human being means a person who has been born and is alive; Bodily injury means physical pain, illness or any impairment of physical condition; Serious bodily injury means bodily injury which creates a substantial risk of death or which causes serious, permanent disfigurement, or protracted loss or impairment of the function of any bodily member organ; Deadly weapon means any firearm or other weapon, device, instrument, material, or substance, whether animate or inanimate, which in the manner it is used or is intended to be used is known to be capable of producing death or serious bodily injury, Criminal Homicide- A person is guilty of criminal homicide if he purposely, knowingly, recklessly, or negligently causes the death of another human being; criminal homicide is murder, manslaughter or negligent homicide. Murder- criminal homicide constitutes murder when it is committed purposely or knowingly; or it is committed recklessly under circumstances manifesting extreme indifference to the value of human life. Such recklessness
13

and indifference are presumed if the actor is engaged or is an accomplice in the commission or, or an attempt to commit, or flight after committing or attempting to commit robbery, rape, or deviate sexual intercourse by force or threat of force, arson, burglary, kidnapping or felonious escape. Murder is a felony of the first degree [but a person convicted of murder may be sentenced to death]. Premeditation- is the essential element which distinguishes first-degree murder from second-degree murder. It is more than a mere intent to kill; it is a fully formed conscious purpose to kill. This purpose to kill may be formed a moment before the act but must exist for a sufficient length of time to permit reflection as to the nature of the act to be committed and the probable result of that act. While premeditation may be proven by circumstantial evidence, the evidence relied upon by the state must be inconsistent with every other reasonable inference. Where the states proof fails to exclude a reasonable hypothesis that the homicide occurred other than by premeditated design, a verdict of first-degree murder cannot be sustained. Merger- old common law rule for same conduct. Misdemeanor merges into a felony. No merger of offenses of same degree. Modern rule- no merger except for solicitation and attempt except for lesser included offenses consists entirely of some but not all elements possession of drugs v. possession of drugs for sale. Jury nullification- jury doesnt believe the law and doesnt follow judges directions. At common law a fetus is not a human, but can be injured, born, then die; that is homicide. Concurrent sufficient cause- both actions could have killed victim.

People v. Thompson (Ariz. 2003) wife got a divorce and 4 bullets as a parting gift from her X. Rule: courts have held that reflection can occur as quickly as successive thoughts of the mind. If not obvious evidence of premeditation exists, the state is allowed to use circumstantial evidence- willful, deliberate and premeditated. Passage of time by itself is not premeditation. Passage of time is 1 factor that can show reflection. It is the act of premeditation and not the length of time available that determines the question. State may not use the passage of time as a proxy for premeditation. State may argue the passage of time suggests premeditation, but it may not argue the passage of time is premeditation. Does the statute abolish the requirement of actual reflection? Whether it eliminates the requirement of direct proof of actual reflection or whether it substitutes for the necessary proof of actual reflection the mere passage of enough time to permit reflection.

Coolen v. State (Fl. 1997) hothead lands in hot water after lots of drinking Rule: premeditation can be shown by circumstantial evidence. Whether or not the evidence shows a premeditated design to commit a murder is a question of fact for the jury.

State v. Doub, III (Kan. App. 2004) crack head kills little girl and a lot of property damage Rule: evidence used in light most favorable to prosecution in appeals. Depraved heart murder is recklessness and extreme indifference to the value of human life, conscious disregard of the risk. Recklessness can be
14

assimilated to purpose or knowledge is treated as depraved heart 2nd degree murder and less extreme recklessness is punished as manslaughter. Malice was eliminated as an element of 2nd degree murder.

MPC Manslaughter- criminal homicide constitutes manslaughter when: it is committed recklessly; or a homicide which would otherwise be murder is committed under the influence of extreme mental or emotional disturbance for which there is reasonable explanation or excuse. The reasonableness of such explanation or excuse shall be determined from the viewpoint of a person in the actors situation under the circumstances as he believes them to be.

People v. Pouncey (Mich. 1991) another hothead blows away a guy for getting in his face Rule: a verdict shall not be set aside where the court fails to instruct on any point of law unless the accused requests such instruction. Murder and manslaughter are both homicide but the element of provocation which characterizes manslaughter is different from murder. Test for voluntary manslaughter- D must kill in the heat of passion. Passion must be caused by an adequate provocation. There cannot be a lapse of time during which a reasonable person could control his passions. The provocation necessary to mitigate a homicide from murder to manslaughter is that which causes the D to act out of passion rather than reason. The provocation must be adequate, that which would cause the reasonable person to lose control. When a jury cannot find provocation was adequate, the judge may exclude evidence of provocation. Did trial judge err in his instruction?

MPC Negligent Homicide- criminal homicide constitutes negligent homicide when it is committed negligently. Negligent homicide is a felony of the 3rd degree.

State v. Williams (Wash. App. 1971) stupid Indians let their kid die because theyre stupid Rule: 2 basic issues- existence of the duty furnish medical aid charged by the information to be violated and the seriousness of the breach required and the issue of proximate cause whether Ds were put on notice in time to save the childs life, that medical care was required. Caring for a minor child is a natural duty. In the case of involuntary manslaughter the breach had to amount to more than mere ordinary or simple negligence- gross negligence was essential. Simple or ordinary negligence describes a failure to exercise the ordinary caution necessary to make out the defense of excusable homicide. In this case D is not father or adopted father of victim but he assumed responsibility for the care of the child; a duty to furnish necessary medical care. One must use ordinary caution when attending to a childs illness; response does not need to be immediate. Reasonable standard should apply to timeliness of seeking a doctor.

Week 6 9/20 and 9/22 Felony Murder, Rape Felony Murder Rule- if a person causes the death of another during the course of a felony, or an attempt to commit the felony, then the killing constitutes murder. In a jurisdiction that divides murder into degrees, this type of murder would be in the second degree, although a number of states provide in their homicide statutes that killings during certain specified felonies raise the charge to 1st degree murder. To be held liable for the killing, it does not matter whether the death was intentional, and the government need not prove any level of
15

mens rea for the killing, just the intent for the underlying felony (or attempted felony) that led to the death. Even completely unintended or accidental deaths can trigger liability if the killing was during the course of an in furtherance of a felony. Malice aforethought is inferred from the intent to commit the underlying felony. The only limitation on the type of felony that may serve as an underlying felony for a felony murder conviction is that the felony must be inherently dangerous to human life. a felony is inherently dangerous when it is dangerous per se or by its circumstances creates a foreseeable risk of death. Limitations- enumerated crimes or inherently dangerous crimes. In furtherance of- in some jurisdictions. Causal link between felony and death. Merger- when the underlying felony involves the intent to do serious bodily harm.

Hines v. State (Ga. 2003) is that a turkey or my buddy, whatever, Im going to shoot at it anyway Rule: depending on facts possession by convicted felon can be an inherently dangerous felony. His illegal possession created a foreseeable risk of death.

State v. Contreras (Nev. 2002) sticks and steal can break your bones and also kill you Rule: Burglary can escalate a homicide to 1st degree murder without necessity of proving premeditation and deliberation. Merger is not appropriate for felony murder when the underlying felony is burglary, regardless of the intent of the burglary. The predicate felony must be one not having the purpose of inflicting (or knowingly inflicting) bodily harm as an element.

State v. Sophophone (Kan. 2001) burglar gets charged with murder while sitting in a cop car Rule: time, distance, and the casual relationship between the underlying felony and a killing are factors to be considered in determining whether the killing occurs in the commission of the underlying felony and the defendant is therefore subject to the felony-murder rule. Agency- majority view, limits application to those homicides committed by the felon or an agent of the felon. The identity of the killer becomes the threshold requirement for finding liability under felony murder doctrine. Proximate cause- liability attached for any death proximately resulting from the unlawful activity- even the death of a co-felon, notwithstanding the killing was by one resisting the crime. Felony murder may preclude consideration of the deceaseds identity, which would make D liable for all deaths caused by others during the crime- minority view. The minority of the state whose courts have adopted the proximate cause theory believe their legislatures intended that any person, co-felon, or accomplice who commits an inherently dangerous felony should be held responsible for any death which is a direct and foreseeable consequence of the actions of those committing the felony. These courts apply the civil law concept of proximate cause to felony murder situations. Criminal statutes are strict liability offenses. Rape- carnal knowledge of a woman forcibly and against her will. The victim must report the incident promptly, corroboration of the victims story, and utmost resistance, heightened the proof necessary for a conviction. Generally, conviction rested on the last of these requirements: how well the woman conveyed her unwillingness to engage in intercourse.

16

MPC Rape- a male who has sexual intercourse with a female not his wife is guilty of rape if: he compels her to submit by force or by threat of imminent death, serious bodily injury, extreme pain or kidnapping, to be inflicted on anyone; or he has substantially impaired her power to appraise or control her conduct by administering or employing without her knowledge drugs, intoxicants or other means for the purpose of preventing resistance; or the female is unconscious; or the female is less than 10 yrs old. Rape is a felony of the 2nd degree unless in the course thereof the actor inflicts serious bodily injury upon anyone, or the victim was not a voluntary social companion of the actor upon the occasion of the crime and had not previously permitted him sexual liberties, in which cases the offense is a felony of the 1st degree. Rape is a general intent crime and mistake of fact is a defense at common law for general intent crimes only if the mistake was both honest (the defendant is not lying in saying he was mistaken) and unreasonable, it means that D can be guilty only if his mistake was unreasonable, that is, negligent. The injury of an unwanted sexual intrusion is great, and there is a clear need to give the man a reason to pay attention to the womans wishes. The code requires negligence + conscious awareness of the risk. Although this is a lesser standard than knowing that the woman did not consent, a D should have known better but was not consciously aware even of the risk of non-consent must be acquitted. Gross Sexual imposition- a male who has sexual intercourse with a female not his wife commits a felony of the 3rd degree if: he compels her to submit by any threat that would prevent resistance by a woman of ordinary resolution; or he knows that she suffers from a mental disease or defect which renders her incapable of appraising the nature of her conduct; or he knows that she is unaware that a sexual act is being committed upon her or that she submits because she mistakenly supposes that he is her husband. Deviate sexual intercourse by force or imposition By force or its equivalent- a person who engages in deviate sexual intercourse with another person or who causes another to engage in deviate sexual intercourse, commits a felony of the 2nd degree if: he compels the other person to participate by force or by threat of imminent death, serious bodily injury, extreme pain or kidnapping, to be inflicted on anyone; or he has substantially impaired the other persons power to appraise or control his conduct, by administering or employing without the knowledge of the other person drugs, intoxicants or other means for the purpose of preventing resistance; or the person is unconscious; or the other person is less than 10 yrs. old. By other imposition- a person who engages in deviate sexual intercourse with another person, or who causes another to engage in deviate sexual intercourse, commits a felony of the 3rd degree if: he compels the other person to participate by any threat that would prevent resistance by a person of ordinary resolution; or he knows that the other person suffers from a mental disease or defect which render him incapable of appraising the nature of his conduct; or he knows that the other person submits because he is unaware that a sexual act is being committed upon him. Corruption of minors and seduction- a male who has sexual intercourse with a female not his wife, or any person who engages in deviate sexual intercourse or causes another to engage in deviate sexual intercourse is guilty of an offense if: the person is less than 16 yrs. old and the actor is at least 4 years older than the other person; or the other person is less than 21 yrs. old and the actor is his guardian or otherwise responsible for general supervision of his welfare; or the other person is in custody of law or detained in a hospital or other institution and the actor has supervisory or disciplinary authority over him; or the other person is a female who is induced to participate by a promise of marriage which the actor does not mean to perform. The first part is a 3rd degree felony. Otherwise the offenses are misdemeanors.

17

Sexual assault- a person who has sexual contact with another not his spouse, or causes such other to have sexual contact with him, is guilty of sexual assault, a misdemeanor, if he knows that the contact is offensive to the other person; or he knows that the other person suffers from a mental disease or defect which renders him or her incapable of appraising the nature of his or her conduct; or he knows that the other person is unaware that a sexual act is being committed; or the other person is less than 10 yrs. old; or he has substantially impaired the other persons power to appraise or control his or her conduct, by administering or employing without the others knowledge drugs, intoxicants or other means for the purpose of preventing resistance; or the other person is less than 16 yrs old and the actor is at least 4 yrs. older than the other person; or the other person is less than 21 yrs. old and the actor is his guardian or otherwise responsible for general supervision of his welfare; or the other person is in custody of law or detained in a hospital or other institution and the actor has supervisory or disciplinary authority over him. Sexual contact is any touching of the sexual or other intimate parts of the person for the purpose of arousing and gratifying sexual desire.

People v. Griffin (Cal. 2004) asshole molested and rapes girl and tries to blame it on her Rule: force has a special legal definition the average juror would not know. Conviction of forcible lewd acts on a minor requires evidence of physical force substantially different from or substantially greater than that necessary to accomplish the lewd act itself. Although resistance is no longer the touchstone of the element of force, the reviewing court still looks to the circumstance of the case, including the presence of verbal or nonverbal threats, or the kind of force that might reasonably induce fear in the mind of the victim, to ascertain sufficiency of the evidence of a conviction [of forcible rape]. There is considerable difference between lewd acts by force on a child under age 14 and the crime of forcible rape. Evidence used in light most favorable to prosecution. Pg. 353, 354- further explanations on force. Did D use force to accomplish intercourse with victim against her will, not whether the force he used overcame victims physical strength or ability to resist him?

NEW STYLE! Week 7 9/27 and 9/29 Rape, theft and property offenses Objective standard- what was actually in her mind doesnt matter. We evaluate it all by what she said and did. Subjective standard- are we looking into the mind of the victim? Can we know what is or was in her mind? Mens rea- MPC silent on mens rea. Must have at least recklessness- conscious disregard of substantial risk. Common law- general intent crime. Negligence is enough (negligent as to whether victim was consenting. Most state statutes follow this. Force- historically, victims resistance was required. Fear- enough if it places victim in fear of great and immediate harm. Incapable of consent- unconscious, drugs or other intoxicants, mental condition.

18

US v. Arab (US Army court of crim. Appeals 2001) crazy motherfucker rapes his wife Rule: consent may determine whether a particular touching is offensive, an honest and reasonable mistake of fact as to the victims consent to being touched may be a defense to this aspect of a touching. Where there is no actual consent, but the activity is one that, with consent, would be lawful, an honest and reasonable belief that the person touched was consenting does not, however, render what would otherwise be a battery lawful under all circumstances. Was consent, if any, if such consent could form the basis for a defense, whether victim actually consented and if she did not consent, whether the government proved beyond a reasonable doubt the appellant was not honestly and reasonably mistaken about victims lack of consent. Consent is no defense when assault involves a means likely to produce death or grievous bodily harm. The test is whether the force used carries a substantial risk of SBI and actual harm need not be demonstrated. No one can consent to assault and battery.

State v. Vander Esch (Iowa App. 2002) dont eat pizza at this place! Rule: sexual abuse- the act is done by force or against the will of the other. If the consent or acquiescence of the other is procured by threats of violence toward any person or if the act is done while the other is under the influence of a drug inducing sleep or is otherwise in a state of unconsciousness, the act is done against the will of the other. Legislative intent governs statutory interpretation. All circumstance (subjective and objective) should be considered. Fraud in factum is rape; fraud in inducement is not rape.

State v. Martinez (Utah 2002) girl said she was legal, oops, mistake is no defense Rule: mistake is not a defense as to the victims age in a prosecution for unlawful sexual intercourse. It is a strict liability offense. Statute does not have a mental state requirement but the commission of the sexual act itself is sufficient to violate to the statute. History of sex with D is always admissible. History of sex with others inadmissible. Reputation for unchastity inadmissible.

Start Bible Here I. Theft and Property Crimes- Larceny (*Capital Offense at Common Law) a. Elements i. A trespassory taking ii. Carrying away of (Asportation) iii. Personal property iv. Of another v. With intent to permanently deprive b. Caption i. Assuming/Obtaining Control Over ii. Mere movement and/or destruction is not enough iii. May achieve caption through an innocent agent (hiring someone to pick something up for you) c. Asportation- A small movement toward taking it away is enough d. Larceny does NOT include taking the following items
19

e. f. g. h.

i.

j.

i. Realty ii. Unsevered Fixtures iii. Services iv. Intangibles v. DOES include gas and Electricity **Possession is more than custody- greater scope of control and authority over i. Employees normally only have custody UNLESS they have been granted broad powers Lost or Mislaid property is considered to be in the constructive possession of the owner Abandoned property is in no ones possession Larceny by Trick- the acquisition of consent by a false promise constitutes a breach of possession. When the consent to the taking is given only because it was included by a misinterpretation. Mens Rea of Larceny i. YES a larceny if: 1. Creates a substantial risk of loss 2. Intent to sell back to owner ii. NOT a larceny if: 1. Intent to borrow 2. Intent to obtain repayment of debt * 7 main separate Theft and Property Crimes* i. Larceny ii. Larceny by Trick iii. Larceny after Trust iv. Embezzlement v. False Pretense vi. Shoplifting vii. Receiving Stolen Property- receiving possession and control of stolen property, known to have been obtained in a manner constituting a criminal offense, by another person, with the intent to permanently deprive the owner of his interest in the property.

Temporarily possessing than intending to possess permanently later is larceny. Stealing from a thief is illegal. In criminal cases thief 1 and 2 are both liable to the government/state. Creating substantial risk of loss, intent to sell back to owner not larceny if- intent to borrow, intent to obtain repayment of debt. May be larceny if- intent to pay, intent to claim reward.

Bell v. US (US 1983) stolen check Rule: although taking and carrying away is not a necessary element of the crime, it is consistent with false pretenses. Mere possession is part of larceny. Common law larceny was limited to thefts of tangible personal property. It excluded thefts of, i.e. - checks. Fed. Statute covers anything over $100. It also expands theft to property not in physical possession of owner. The statute is not limited to common law larceny and although it may not cover the full range of theft offenses, it covers Ps conduct.
20

Whoever takes and carries away, with intent to steal or purloin, any property or money or any other thing of value exceeding $100 belonging to, or in the care, custody, control, management, or possession of any bank, credit union, or any savings and loan association, shall be fined not more than $5g or imprisoned not more than 10 years, or both.

People v. Shannon (Cal. 1998) did he steal the clothes or not? Rule: theft- the unlawful taking of anothers property. The crime includes larceny, embezzlement, larceny by trick, and theft by false pretenses. Larceny, larceny by trick, and embezzlement involve taking anothers personal property from the owners possession, without the owners consent, with the intent to deprive the owner permanently of the property. Theft by consent, with the intent to deprive the owner permanently of the property; it requires that the D use false pretenses to induce the other to give the property to him. D does not need to remove the clothes from the store to satisfy asportation. D only needs to move it slightly with intent to deprive owner permanently. Intent to restore or make restitution of property is no defense. D does not need to take for his own use; deprivation of possession is all that matters.

People v. Stay (Cal. 1971) shopping cart pirate Rule: although D never intended to keep carts, but stole them intending to ransom them back to owners or other stores, he completed the thefts.

US v. Mafnas (9th Cir. 1983) armored truck robbery Rule: court held that statute applies only to common law larceny which requires a tresspassory taking. WRONG! Custody is temporary, possession is permanent. Taking the money was beyond the owners (banks) consent. D was not an agent/bailee. Embezzlement- fraudulent conversion of property (usually same as larceny) of another by a person in lawful possession with the property entrusted to him or her. A Bailee who breaks bulk commits larceny. The Bailee-carrier was given possession of a bale, but not its contents. Therefore, when the Bailee pilfered the entire bale, he was not guilty of larceny; but when he broke open the bale and took a portion or all of the contents. He was guilty of larceny because his taking was trespassory and it was from the constructive possession of another. I. Embezzlement a. Elements i. Fraudulent ii. Conversion iii. Of Property (Usually same as Larceny) iv. Of Another v. By a Person in Lawful Possession vi. With the Property entrusted to him or her

21

State v. Parris (SC App. 2003) mobile home deals goes sour Rule: breach of trust with fraudulent intent- larceny after trust, which includes all of the elements of larceny or in common parlance, stealing, except the unlawful taking in the beginning. The fact distinguishing larceny from breach of trust is that possession of the property is gained by unlawful means in larceny while a breach of trust is accomplished by a lawful taking of the property- through its entrustment to one by another. A trust is an arrangement whereby property is transferred to another with the intent that it be administered by the trustee for the benefit of the transferor or a 3rd party. It is a fiduciary relationship which arises as a result of a manifestation of an intention to create it. A trust relationship is created by the express intent to do so, either through words or conduct. A trust may arise by implication in the absence of an express common intent if there is a fiduciary relationship between the parties, because a person in a fiduciary relation to another is under a duty to act for the benefit of the other as to matters within the scope of the relation. The ordinary relationship between unaffiliated parties is not fiduciary. A fiduciary relationship may be created when one reposes special confidence in another so that the latter, in equity and good conscience, is bound to act in good faith and with due regard to the interests of the one reposing the confidence. Facts and circumstances must indicate the party reposing trust in another has some foundation for believing the one so entrusted will act not in his own behalf but in the interest of the party so reposing. State needs to establish trust relationship without which there will be a directed verdict of acquittal.

Week 8 10/4 and 10/6 Theft and property offenses continued and midterm State v. Jennings (Ohio App. 2002) receiving stolen property Rule: no on shall receive, retain, or dispose of property of another knowing or having reasonable cause to believe the property has been obtained through commission of a theft offense. A $ card is a credit card. Expired cards fit the statutory definition.

I.

II.

False Pretense a. Elements i. Obtaining title ii. To the property of another iii. By an intentional (or knowing) false settlement of past of existing fact iv. With the intent to defraud the other v. *Note: Title passes here unlike in larceny and embezzlement b. Notes on False Pretense i. Nondisclosure is not enough ii. Representation must actually be false iii. Opinions/Puffing (Sales Pitch) is NOT enough iv. Promises for the future are not enough 1. *But MPC and statutes now recognize deception as to state of mind 2. Requires proof of broken promises v. If youre getting your own property back, NOT false pretense because theres no intent to defraud. Receiving Stolen Property a. MPC does not say if Mens Rea needed- so it must be assumed at least recklessly.
22

III.

b. At common law, normally knowing c. Possession i. Need not be manual possession ii. May designate a place to drop it off Robbery a. Elements (First 3 elements of larceny, PLUS) i. Trespassory taking and carrying away of ii. Personal property of another iii. With the intent to permanently deprive iv. From others person or presence v. Proximity and Control vi. By force or intimidation 1. Threat of immediate serious harm to person is required (MPC); common law allows threat of dwelling 2. If not immediate or physical, crime is EXTORTION (or blackmail)

Commonwealth v. Powell (Mass. 2001) dangerous weapon or not? Rule: armed robbery- D must be armed with dangerous weapon or items that are used or displayed in a way such that they reasonably appear capable of causing serious injury or death. Even an object on close inspection is incapable of inflicting serious injury or death, can still be a dangerous weapon if at the time of the offense it would have been reasonable to believe that it was capable of inflicting injury. If a robber said he had a gun and never actually did, convicting for armed robbery is not warranted. Being apparently armed is not enough. Just saying you have a gun is not enough. Victims view of the nature of the item governs, or if it is dangerous.

Commonwealth v. Jones (VA 2004) shoe thief is packing heat Rule: robbery, common law- the taking, with intent to steal of the personal property of another, from his person or in his presence, against his will, by violence or intimidation. Violence or intimidation must occur before or at the time of the taking. As Ds larceny continued but before his custody was converted into possession, manager of store intervened. When D introduced force and violence by producing a gun- his crime transformed into robbery. I. Burglary a. Elements i. Breaking and entering (actus reus) ii. The dwelling house (attendant circumstances) iii. Of Another (attendant circumstances) iv. At nighttime (attendant circumstances) v. With the intent to commit a felony therein (intent)

23

State v. Crossman (ME 2002) nosey neighbors catch a robber Rule: burglary- entering or surreptitiously remaining in a structure, with knowledge that the actor is not licensed or privileged to do so and with the intent to commit a crime in the structure. A burglarious entry is accomplished by the intrusion into the building of any part of the body, providing the instrument is inserted and utilized as a means of effectuating or attempting to effectuate the theft and not solely as a means of accomplishing the breaking into the building. Prior cases have found a breaking and entering where D merely moved to a material degree something that barred the way, either a closed door or window. Common law burglary- the breaking and entering of a dwelling if another in the nighttime with intent to commit a felony therein. It is also necessary that the state prove that the accused had the intent to break and enter in addition to the intent to commit a felony therein. It is for this reason that common law burglary is considered a specific intent crime. Burglary requires the specific intent to commit a crime upon entry, and that intent must be formed before the burglary.

After Midterm Week 9 10/11 and 10/13 Attempt: actus reus & mens rea I. Attempt a. Common Law- Two elements required for conduct to be deemed a criminal attempt: i. Intent to commit the crime alleged attempted; ii. Some acts in furtherance of that intent b. Attempt in two types: Complete and Incomplete Attempt i. Complete: defendant does everything possible to effect the crime but does not cause the result of the particular offense, (killing victim ii. Incomplete: defendant stops short of completing the offense, either of his own accord or because of the intervention of a third party c. Steps to Completing a Crime i. Conceiving idea ii. Evaluate idea- whether to proceed iii. Forms the intention to do it iv. Prepare- acquires instruments and information v. Commence the commission of the offense vi. Completing the actions required. d. The Overt Act (Actus) i. Difference between (mere) preparation and perpetration: a defendant is not guilty of an attempt when the conduct has not gone beyond preparation. ii. Approach Tests 1. Proximity Approach- the act must be sufficiently proximate to the intended crime. The strictest approach is that the accused must have engaged in the last proximate act. 2. The Probable Desistance Approach: contemplates an act which in the ordinary course of events would result in the commission of the intended crime except for the intervention of some extraneous factor. Under this approach the accuseds
24

conduct must pass that point where most men, holding such an intention as the accused holds, would think better of their conduct and desist. 3. The Equivocality Approach: the act which transforms the accuseds conduct from preparation to perpetration constitutes a step towards the commission of the intended crime, and the doing of the act can have no other purpose than the commission of that crime. 4. The Model Penal Code Approach: a person is guilty of an attempt to commit a crime if acting with the kind of culpability otherwise required for commission of the crime, he purposely does or omits to do anything which, under the circumstances as he believes them to be, is an act or omission constituting a substantial step in a course of conduct planned to culminate in his commission of the crime. e. Criminal Attempt (MPC) 1. Definition of Attempt: A person is guilty of an attempt to commit a crime if, acting with the kind of culpability otherwise required for commission of the crime, he: a. Purposely engages in conduct that would constitute the crime if the attendant circumstances were as he believes them to be; or b. When causing a particular result is an element of the crime, does or omits to do anything with the purpose of causing or with the belief that it will cause such result without further conduct on his part; or c. Purposely does or omits to do anything that, under the circumstances as he believes them to be, is an act or omission constituting a substantial step in a course of conduct planned to culminate in his commission of the crime. f. Mens Rea of Attempt i. Since general intent typically will not suffice, many jurisdictions hold that attempt cannot be used when the object crime is defined merely by recklessness or negligence. For a specific intent crime, the government will have to show both the intent for the offense and the intent to commit the offense. ii. Common Law (3 requirements) 1. Having whatever mental state the complete crime requires 2. Having the purpose to bring about the acts and any results that there are elements of the completed crime 3. Merely knowing of the existence of any attendant circumstances that the completed crime requires.

g. Criminalizing Preparatory Conduct i. An attempt charge requires the government to prove criminality based on what can be inferred from the evidence regarding how a course of events would have developed and what the defendants state of mind was in pursuing the conduct. A state legislature can make certain conduct a crime that is preliminary to another offense. (Making a threat to kill someone.
25

h. Impossibility i. Factual Impossibility- The defendant is mistaken about the facts, but if the defendant had been correct the crime could be accomplished (Attempting to steal from an empty pocket). The defendant is held accountable for the attempted crime because this factual impossibility does not negate the defendants intent to commit the crime. ii. Legal Impossibility- Legal Status of the crime (attendant circumstance) does not exist (killing a dummy; receiving stolen goods that were not stolen. 1. True Legal Impossibility- When even if the object of the crime had been accomplished there would be no crime, the defendant cannot be held criminally liable. i. Abandonment and Renunciation i. Voluntary abandonment is an affirmative defense. It is an affirmative defense to a charge of criminal attempt, solicitation, or conspiracy that the person, after committing the criminal attempt, solicitation, or conspiracy, prevented the successful commission of the offense attempted, solicited, or conspired, under circumstances manifesting a complete and voluntary renunciation of the persons criminal purpose. ii. Renunciation is not complete if it is motivated by a decision to postpone the criminal conduct until a more advantageous time or to transfer the criminal effort to another but similar objective or victim.

Common law criminal attempt- intent to commit the crime alleged attempted, some acts in furtherance of that intent. While it was generally agree that something more than mere criminal intent was necessary and that some overt act was required, a variety of approaches was used to determine what acts would satisfy the requirements. A common purpose of all the approached was to provide a basis for determining whether the accused has done enough to justify an interference of a criminal intent and to give rise to attempt liability. Complete attempt- D does everything possible to effect the crime but does not cause the result of the particular offense- killing the victim. Incomplete attempt- D stops short of completing the offense, either of his own accord or because of the intervention of a 3rd party. The law treats them identically as far as proving the crime and imposing punishment. The distinction may be important for determining what defenses are available to the D. For a complete attempt, D is most likely to argue that the conduct is insufficient to show his intent to commit the alleged crime. For an incomplete attempt, D can argue that his conduct is insufficient to show that he had moved beyond the preparatory stage to actually attempting to commit the crime. Classic distinction between mere preparation and preparation- a D is not guilty of an attempt when the conduct has not gone beyond preparation. Pg. 429-430 approaches to testing attempt. Proximity, probable desistance, equivocality, and MPC approacha person is guilty of an attempt to commit a crime if, acting with the kind of culpability otherwise required for commission of the crime, he purposely does or omits to do anything which, under the circumstances as he believes them to be, is an act or omission constituting a substantial step in a course of conduct planned to culminate in his commission of the crime.
26

MPC Criminal Attempt pg. 433-4 There must be an overt act to have liability, mere preparation is not enough. Act must be a substantial step. Attempt always merges with target offense. Substantial step is primary test now, it must be strongly corroborative of the actors criminal purpose.

State v. Stewart (Wisc. 1988) beggar begs a bit too much Rule: an attempt to commit a crime requires that the actor have an intent to perform acts and attain a result which, if accomplished, would constitute such crime and that he does acts toward the commission of the crime which demonstrate unequivocally, under all circumstances, that he formed that intent and would commit the crime except for the intervention of another person or some other extraneous factor. To prove an attempt, the state must prove an intent to commit a specific crime accompanied by sufficient acts to demonstrate unequivocally that it was improbable the accused would desist of his or her own free will. The intervention of another person or some other extraneous factor that prevents the accused from completing the crime is not an element of the crime of attempt. If the individual, acting with requisite intent, commits sufficient acts to constitute an attempt, voluntary abandonment of the crime after that point is not a defense. 2 elements of attempt- an intent to commit the crime charged and sufficient acts in furtherance of the criminal intent to demonstrate unequivocally that is was improbable the accused would desist from the crime of his or her own free will. In order for D to be found guilty of attempted robbery it must be shown that Ds actions in furtherance of the crime clearly demonstrate, under the circumstances that he had the requisite intent to commit the crime of attempted robbery, and that having formed such intent D had taken sufficient steps in furtherance of the crime so that it was improbable that he would have voluntarily terminated his participation in the commission of the crime. Intent may be inferred from Ds conduct, including words and gestures taken in the context of the circumstances. When a person desists from acts that appear criminal, the intent of the actor may appear equivocal. Desistance thus raises a factual question relevant to the element of intent. It is primarily the acts of the accused which provide evidence of the requisite mental intent . . . the acts must . . . establish that the accused intended to commit the substantive crime. The acts of the accused must not be so few or of such an equivocal nature as to render doubtful the existence of the requisite criminal intent.

Evans. v. State (Ga. App. 1995) casing a parking lot for cars is a crime Holding: discussion about theft and possession of tools is not an attempt to enter, rather they were preparatory acts not proximately leading to the consummation of the crime. They went beyond preparing when they drove to shopping center in search of specific cars. Rule: a person commits the offense of criminal attempt when, with intent to commit a specific crime, he performs any act which constitutes a substantial step toward the commission of that crime. In order to constitute the offense of attempt to commit a crime, the accuse must do some act towards its commission. Commission means the act of committing, doing, or performing; the act of perpetrating. Mere acts of preparation, not proximately leading to the consummation of the intended crime, will not suffice to establish an attempt to commit it. To constitute an attempt there must be an act done in pursuance of the intent, and more
27

or less directly tending to commission of the crime. In general, the act must be inexplicable as a lawful act, and must be more than mere preparation. Yet it cannot accurately be said that no preparations can amount to an attempt. The fact that further steps must be taken before the crime can be completed does not preclude such a finding that the steps already undertaken are substantial.

Threat- is an expression of an intention to inflict evil, injury, or damage on another. Mens Rea- an attempt charge requires the government to prove D acted with specific intent to commit the target offense. Since general intent typically will not suffice, many jurisdictions hold that attempt cannot be used when the object crime is defined merely by recklessness or negligence. For a specific intent crime, the government will have to show both the intent for the offense and the intent to commit the offense. Proof of intent- can be based on circumstantial evidence. For an attempt crime, it will often be difficult to determine what D intended when the attempt is incomplete. When D completes 1 crime, that evidence may not be sufficient to establish the intent to commit a more serious crime.

Baldwin v. Commonwealth (Va. 2007) dont run from the cops once theyve stopped you Rule: while a person may be guilty of murder though there was no actual intent to kill, he cannot be guilty of an attempt to commit murder unless he has a specific intent to kill.

3 mens rea requirements- having whatever mental state the completed crime requires, having the purpose to bring about the acts and any results that are elements of the completed crime, and merely knowing of the existence of any attendant circumstances that the completed crime requires.

Week 10 10/18 and 10/20 Attempt: impossibility, abandonment and solicitation and conspiracy Pg. 447 factual and legal impossibility. Pg. 448 true and legal impossibility. Common law recognized only legal impossibility. MPC recognizes only true legal impossibility. Neither factual no hybrid legal impossibility is defense to attempt. MPC- only defense is true legal impossibility. Solicitation- common law- target offense: felony. Request must be received. Solicitation can be a felony. No defense of renunciation. Impossibility never a defense. MPC- target offense: any crime. Request need not be received. Solicitation is a felony of X. Renunciation is a defense if it stops the crime.

Chen v. State (Tex. Crim. App. 2001) internet sex with kid who does not exist Rule: the distinction between factual and legal impossibility has been characterized as turning on whether the goal of the actor was deemed by the law to be a crime. Due to a factual condition unknown to appellant (girl did not exist), the offense of sexual performance by a child could not be completed. It is true that, as appellant claims, the actual offense of sexual performance by a
28

child would have been impossible for appellant to complete; the complainant, girl, did not physically exist. But completion of the crime was apparently possible to appellant. He had specific intent to commit the offense of sexual performance by a child, and he committed an act amounting to more than mere preparation that tended but failed to effect the commission of the offense. Legal impossibility exists where the act if completed would not be a crime, although what the actor intends to accomplish would be a crime. It also exists when what the actor intends to do would not constitute a crime, or at least the crime charged. Factual impossibility exists when due to a physical or factual condition unknown to the actor, the attempted crime could not be completed. It refers to a situation in which the actors objective was forbidden by the criminal law, although the actor was prevented from reaching that objective due to circumstance unknown to him. Where D is charged with an attempt to commit a crime it is immaterial whether the attempted crime is impossible of completion if, as in the present case (cited case), completion was apparently possible to D who was acting with the intent to commit the crime of murder. Whether a 47yr old male undercover cop posing as a 13yr old female for the purposes of internet communications was sufficient for conviction. Girl did not exist so D said state failed to prove case.

Common law abandonment- may be evidence that there is insufficient conduct to prove an attempt. It can also be used to infer that the accused did not have the requisite intent to attempt the object offense. It is not considered as a separate defense to the charge of an attempted crime. MPC renunciation- it is an affirmative defense to a charge of criminal attempt, solicitation or conspiracy that the person, after committing the criminal attempt, solicitation or conspiracy, prevented the successful commission of the offense attempted, solicited or conspired, under circumstances manifesting a complete and voluntary renunciation of the persons criminal purpose. Renunciation of criminal purpose is not voluntary if it is motivated, in whole or in part, by circumstances, not present or apparent at the inception of the actors course of conduct, that increase the probability of detection or apprehension or that make more difficult the accomplishment of the criminal purpose. Renunciation is not complete if it is motivated by a decision to postpone the criminal conduct until a more advantageous time or to transfer the criminal effort to another but similar objective or victim. As an affirmative defense, D bears the burden of proof by a preponderance of the evidence to establish the voluntary renunciation of the attempted crime. It is necessary for D to show that the abandonment is both voluntary and complete.

Patterson v. State (Ind. App. 2000) broke window to house but changed mind as to entering Rule: P will not be liable if subsequent to substantial step towards committing crime but prior to its consummation, he voluntarily abandoned his efforts. Decision to abandon must originate with D.

Threat rule- whoever transmits in interstate or foreign commerce any communication containing any threat to kidnap any person or any threat to injure the person of another.
29

State v. McCarthy (Mt. 1999) restraining orders and stalkers Rule: stalking occurs when the stalker repeatedly harasses, threatens, or intimidates the stalked person in person, or by phone, by mail, or by other action, devise or method. Communicating through a 3rd party whom the stalker knows is likely to relay the fact of contact, and hence produce the desired effect of harassing or intimidating the victim, constitutes and action, device or method of stalking.

I.

Solicitation a. Definition i. The crime of asking another person to join in a course of criminal conduct, with the intent that the other person commit the crime or participate in its commission. b. Solicitation must be distinguished from the situation in which one person uses another to commit a crime unbeknownst to the other person c. Solicitation is similar to a criminal attempt because it is a preliminary step toward the completion of a crime that usually does not come to fruition. d. Criminal Solicitation (MPC) i. Definition: A person is guilty of solicitation to commit a crime if with the purpose of promoting of facilitating its commission he commands, encourages, or requests another person to engage in specific conduct that would constitute such crime or an attempt to commit such crime or would establish his complicity in its commission or attempted commission. ii. Uncommunicated Solicitation: It is immaterial under Subsection (1) of this section that the actor fails to communicate with the person he solicits to commit a crime if his conduct was designed to effect such communication iii. Renunciation of Criminal Purpose: It is an affirmative defense that the actor, after soliciting another person to commit a crime, persuaded him not to do so or otherwise prevented the commission of the crime, under circumstances manifesting a complete and voluntary renunciation of his criminal purpose.

Solicitation- is the crime of asking another person to join in a course of criminal conduct, with the intent that the other person commit the crime or participate in its commission. It must be distinguished from the situation in which one person uses another to commit a crime unbeknownst to the other person. It is similar to a criminal attempt because it is a preliminary step toward the completion of a crime that usually does not come to fruition. Nothing more need happen once the person asks another to commit the crime. It is prepatory to the crime of conspiracy. It merges into the crime of attempt and the completed offense. If the government cannot prove the attempt but has evidence that D asked another person to engage in criminal conduct, then that can be the basis for a criminal conviction. It is usually the case that a solicitation charge will be drought when the target offense has not been completed, which frequently occurs for 2 reasons. The solicitation was not received by the recipient. The issue then is whether an undelivered (or intercepted) solicitation should constitute a crime. When the solicitation is made to an undercover police officer or person who has no intention of carrying out the crime, the object of the solicitation will never occur. It is usually punished less severely than an attempt. If other person reasonably believes he is helping- not committing a crime, that person is not liable. Pg. 466 MPC Criminal Solicitation
30

People v. Saephanh (Cal. App. 2000) letters to buddy from jail to kill wife Rule: Cal. Said solicitation requires a completed communication- not just intent . . . putting letter in mail and possessing the requisite criminal intent. Un-communicated soliciting messages do not expose others to inducements to commit crimes. Nor is there a likelihood that an un-communicated message would result in the commission of crimes. Letters posted but not delivered do not give rise to the dangers from which section 653f seeks to protect society. Section 653f- requires that 2 or more persons must be involved, at least 1 being necessarily a solicitor and the other necessarily being the person solicited. Section 664- Every person who attempts to commit any crime, but fails, or is prevented or intercepted in its perpetration, shall be punished where no provision is made by law for the punishment of those attempts. Solicitation is a crime, and thus falls within section 664, which applies to the attempted commission of any crime. The legislature is aware of specific provisions regarding attempt in the context of some crimes, and it expressly applies to those crimes which do not address attempt. Not every solicitation is an attempted conspiracy. Solicitation is complete when the solicitation is made, when the soliciting message is received by its intended recipient. It is immaterial that the object of the solicitation us never consummated, or that no steps are taken towards its completing. Unlike assault, which is statutorily defined as an attempted battery. Communication is only completed when it is received by its intended recipient. Attempted solicitation is a crime in Cal.

State v. Disanto (SD 2004) hired hit- man, then backs out Rule: attempt- any person who attempts to commit a crime and in the attempt does any act toward the commission of the crime, but fails or is prevented or intercepted in the perpetration thereof, is punishable. To prove an attempt, therefore, the prosecution must show that D had the specific intent to commit the crime, committed a direct act toward the commission of the intended crime and failed or was prevented or intercepted in the perpetration of the crime. The boundary between preparation and attempt lies at the point where an act unequivocally demonstrates that a crime is about to be committed. The term act presupposes some direct act or movement in execution of the design, as distinguished from mere preparation, which leave the intended assailant only in the condition to commence the first direct act toward consummation of his design. The unequivocal act toward the commission of the offense must demonstrate that a crime is about to be committed unless frustrated by intervening circumstances. This act need not be the last possible act before actual accomplishment of the crime to constitute an attempt. The recognition that solicitation is in the nature of the incitement or encouragement of another to commit a crime in the future and so it is essentially prepatory to the commission of the targeted offense. Acts of mere preparation in setting the groundwork for a crime do not amount to an attempt. To call solicitation an attempt is to do away with the necessary element of an overt act. Pg. 486 MPC Criminal Conspiracy I. Conspiracy
31

a. Requires an agreement between two or more people, although it is not a contract in any legal sense of the term. The agreement does not need to be in writing, does not require specific statements by the parties, and can be ascertained from evidence as slight as the nod of ones head or even what appears to be a concerted activity. A conspiracy can be inferred from the facts and circumstances of the case. They key is that the parties have an understanding as to the common purpose. Knowledge of another persons criminal purpose, without an agreement, however, is insufficient for a conspiracy. b. Elements i. Agreement b/w 2 people ii. Intent to enter an agreement iii. Intent to accomplish some unlawful purpose iv. Most statutes now say purpose must be criminal c. Criminal Conspiracy Elements (MPC) i. Definition- a person is guilty of conspiracy with another person or persons to commit a crime if with the purpose or promoting or facilitating its commission he: 1. Agrees with such other person or persons that they will engage in conduct that constitutes such crime or an attempt or solicitation to commit such crime; or 2. Agrees to aid such other person(s) in planning or commission of such or crime or of an attempt or solicitation to commit such crime ii. Merger- Sometimes occurs in MPC, NOT in CL iii. Overt Act- Required here iv. Agreement- must be a crime v. Intent- purposely not just knowingly 1. Only if you have the purpose to see the act occurring vi. Plurality rule does NOT apply here vii. No Whartons rule or impossibility rule viii. Renunciation effective (Must thwart) d. Court can charge for both Conspiracy an Target Offense i. There is no merger in Common Law (Exceptions in MPC) e. Key Points to Conspiracy i. Mens Rea- MPC Purposely ii. Common Law- purposely can sometimes be inferred from knowledge iii. Need not know co-conspirators 1. But must be aware that other people are involved. f. Plurality Rule: If one is acquitted, the other person cannot be convicted. i. Plurality rule does not apply if more than one person is not agreeing to conspiracy 1. Ex: If there is a person and a cop, there cannot be a plurality because a cop cannot agree to a crime. ii. MPC rejects this rule iii. With CL, husband and wife are ONE PERSON (Old CL) iv. Corporation and its own agent- usually NOT 2 persons g. Whartons Rule: If a crime requires 2 people, its not conspiracy when 2 people agree to do it (Dueling, Adultery).
32

h. Pinkerton Rule: If a co-conspirator decides to do other unlawful crimes as a result of the crime being committed, as long as they are in furtherance of the crime, the other co-conspirators CAN BE LIABLE FOR ADDITIONAL CRIMES THEY DID NOT COMMIT! (Rob a store with toy gun, use real gun, everyone liable; Rob store than set building on fire, not everyone liable, out of the scope of robbery). i. Withdrawal from conspiracy i. Abandonment typically NOT a defense ii. But, you can withdraw, limiting liability for future crimes of conspiracy. 1. Requires informing EVERY member of the conspiracy of you withdrawal. j. Multiple Conspiracies v. Multiple Objects of One Conspiracy i. Huge implications of punishment ii. Is there more than one agreement? iii. RULE: There are as many conspiracies as there are separate agreements iv. Exemption Rule: where the purpose of the Statute is to protect a certain class of individuals, the victim cannot conspire to victimize himself. 1. Ex: Statutory Rape- A child cannot be conspired to commit Statutory Rape on him/himself k. **In sum, the danger which a conspiracy generates is not confined to the substantive offense which is the immediate aim of the enterprise. Conspiracy- is an inchoate crime that protects the public from concerted criminal activity. It requires an agreement between 2 or more people, although it is not a contract in any legal sense of the term. The agreement does not need to be in writing, does not require specific statements by the parties, and can be ascertained from evidence as slight as the nod of ones head or even what appears to be concerted activity. A conspiracy can be inferred from the facts and circumstances of the case. The key is that the parties have an understanding as to the common purpose. Knowledge of another persons criminal purpose, without an agreement, however, is insufficient for a conspiracy. Some conspiracy statutes require an overt act, which is some step toward completing the object of the conspiracy. An overt act need not be illegal in itself. The common law did not require an overt act, but the general federal conspiracy statute has as an element of the offense a conspirator must do any act to effect the object of the conspiracy. A conspiracy charge can reach conduct that occurs at the earliest stages of criminal activity because the crime is complete when the Ds enter into the criminal agreement and commit and overt act (if required). Prosecutors can charge a conspiracy even when the actions of the conspirators never proceed as far as necessary to constitute an attempt to commit the object crime.

US v. Fitz (8th Cir. 2003) Rule: government must prove there was a conspiracy with an illegal purpose, D was aware of the conspiracy, and that he knowingly became a part of it. There must be evidence that D entered into an agreement with at least one other person and that the agreement had as its objective a violation of law. It is not necessary to prove an overt act in furtherance of a conspiracy. The conspiracy may be proved through circumstantial evidence and may be implied by the surrounding circumstances or by inference from the actions of the parties. Once the government establishes the existence of a drug conspiracy, only slight evidence linking D to the conspiracy is required to prove Ds involvement and support the conviction.
33

Conspiracy majority rule- conspiracy is a separate offense from any substantive crimes committed in the course of the conspiracy. Unlike solicitation and attempt, which merge with the target offense, conspiracy can be charged and punished in addition to any punishment for the object of the conspiracy. It is possible for D to be found not guilty of a crime but guilty of a conspiracy to commit that offense because it is the agreement that constitutes the actus reus of conspiracy and not the actual or attempted commission of the substantive crime. Abandonment- since the agreement forms the basis of the crime, if there is an agreement and an overt act in furtherance of the conspiracy- assuming one is needed- then subsequent abandonment is irrelevant to whether the crime occurred. Upon entering into the agreement, and commission of an overt act, if necessary, the crime is complete. If the accused, however, abandons the agreement and withdraws from the conspiracy, avoiding criminal liability may be possible.

Week 11 10/25 and 10/27 Conspiracy continued and accomplice liability US v. Jones (7th Cir. 2004) dont buy too many guns or ATF will be on your ass Rule: in order to establish conspiracy government must prove (1) an agreement to commit an illegal act, (2) Ds knowing and intentional participation in the agreement, and (3) an overt act committed in furtherance of the agreement. Government may rely on circumstantial evidence to establish both the existence of a conspiracy and Ds involvement. But although a jury may infer facts from other facts derived by inference, each link in the chain of inferences must be sufficiently strong to avoid a lapse into speculation. Mere presence while a crime is being committed is insufficient to show that D acted to further a conspiracy. Conspiracy requires specific intent. One must have knowledge of the agreement and also have the specific intent to agree and commit the unlawful objective of the agreement. The government does not, however, have to prove that a conspirator had knowledge of all the circumstances of the government or knew all the participants in the conspiracy. The key is whether there is knowledge of the agreement for the common objective, and if that knowledge can be the basis to infer the requisite specific intent to agree. Willful blindness is good enough to infer D was aware of the conspiracy. Conspiracy is a specific intent crime. Plurality rule- if 1 member is acquitted of conspiracy, the other must be too. MPC rejects this rule. US v. McDermott (2nd. Cir. 2002) insider trading with porn star Rule: in order to prove a single conspiracy, government must show that each alleged member agreed to participate in what he knew to be a collective venture directed toward a common goal. The coconspirators need not have agreed on the details of the conspiracy, so long as they agreed on the essential nature of the plan. The essence of the conspiracy is the agreement and not the commission of the substantive offense. Additionally, it is a long-standing principle that nobody is liable in conspiracy except for the fair import of the concerted purpose or agreement as he understands it; if later comers change that, he is not liable for the change; his liability is limited to the common purposes while he remains in it. Vicarious liability- Williams could be found guilty of possessing the firearm as a felon even if she lacked either actual or constructive possession, as long as another member of the conspiracy possessed a gun. Pinkerton rule- a person may be convicted both for a conspiracy and a substantive offense, and it is not material that overt acts charged in the conspiracy count were also charged and proved as substantive offenses. Second, an overt act of one conspirator may be the act of all without any new agreement specifically directed to that act.
34

The court then considered whether a conspirator could be found guilty of the substantive offense committed by a co-conspirator in furtherance of the conspiracy. It held that the governing principle should be the same where the overt acts in the conspiracy constitute a substantive offense, and that a conspirator could be convicted of the substantive offense committed by a co-conspirator as long as the offense was committed in furtherance of the conspiracy, fell within the scope of the unlawful project, and was reasonably foreseeable as a necessary or natural consequence of the unlawful agreement. The jury must focus on the coconspirators act, on whether it is a crime, on whether the coconspirators guilt of this crime was proved beyond a reasonable doubt, and on whether it was committed in furtherance of the conspiracy in which D participated. Whartons rule- if crime requires 2 people- not conspiracy when 2 people agree to do it.

I.

Accomplice Liability a. A basic principle of accomplice liability is that the liability of one person may be derivative of the criminal conduct of another person. Under this analysis, a person who provides some aid for a crime, such as driving the getaway vehicle from a robbery or shouting encouragement to an assailant, can be convicted of the same offense as the person who actually robs the store or beats the victim. b. Liability for Conduct of Another; Complicity (MPC) i. A person is guilty of an offense if it is committed by his own conduct or by the conduct of another person for which he is legally accountable, or both. c. Conduct Designed to Aid Another in Commission of a Crime: A person who engages in conduct design to aid another to commit a crime that would establish his complicity, if the crime were committed by such other person, is guilty of an attempt to commit the crime, although the crime is not committed or attempted by such other person.

State v. Barnum (Mo. 2000) encouraging a beating is not a good idea Rule: a person is criminally responsible for the conduct of another when either before or during the commission of an offense, with the purpose of promoting the commission of the offense, he aids or agrees to aid or attempts to aid such other person in planning, committing or attempting to commit the offense. This subsection is designed to make individuals who could not be guilty of a crime solely on the basis of their own conduct, guilty nonetheless as an accessory. Accomplice liability- it comprehends any of a potentially wide variety of actions intended by an individual to assist another in criminal conduct. The evidence need not show D personally committed every element of the crime. While mere presence at a crime scene, considered alone or in combination with a refusal to interfere, is insufficient to support a conviction, the broad concept of aiding and abetting plainly encompasses acts that could be construed as encouragement or its derivation. Mere encouragement is enough. Encouragement is the equivalent of conduct that by any means countenances or approves the criminal action of another. Countenances or approves includes encouraging or exciting a criminal act by words, gestures, looks, or signs. Associating with those that committed the crime before, during or after its occurrence, acting as part of a show of force in the
35

commission of the crime, attempting flight from the crime scene, or failing to assist the victim or seek medical help are all factors which may be considered. Missouri eliminated principals and accessories- now all persons who act in concert are equally guilty. Encouragement is participation even if suggestions were not followed. Modern statutory approach eliminates distinction between principals and accessories before the fact.

Liability of one person may be derivative of the criminal conduct of another person. A person who provides some aid for a crime, such as driving the getaway vehicle from a robbery or shouting encouragement to an assailant, can be convicted of the same offense as the person who actually robs the store or beats the victim. Pg. 514 common law accomplice liability Modern statutory approach- whoever commits an offense against the US or aids, abets, counsels, commands, induces or procures its commission, is punishable as a principal. Accessory liability is a means to prove a person committed a crime, but it is not a separate offense that is, D is not guilty of aiding and abetting, the person is guilty of the crime for which he provided assistance or encouragement. Whoever, knowing that an offense against the US has been committed, receives, relieves, comforts or assists the offender in order to hinder or prevent his apprehension, trial or punishment, is an accessory after the fact. One who is an accessory after the fact does not participate in the commission of the primary offense. Instead, an accessory is one who provides assistance to the offender by helping him to avoid apprehension or prosecution after he has already committed an offense. Accordingly, an accessory does not incur liability as a principal. For this reason, the offense of accessory after the fact is considered a far less serious offense than the primary offense, and the accessory, who has not played a role in the actual commission of an offense, is considered far less culpable than one who has participated in the commission of the offense. In fact, the accessory after the fact statute specifically limits the punishment for that offense to no more than half of the punishment prescribed for the primary offense. Government must prove that the person provided some assistance- common terms are aid, abet, counsel, or encourage- in the commission of the offense by the primary violator- the principal- and acted with the requisite intent for the commission of the target offense. Mere presence at the scene of the crime is considered insufficient. In order to aid and abet a crime, a principal must have committed the offense. That does not, however, mean that the principal must be convicted of the crime. Although D may not be convicted of aiding and abetting if the guilt of the principal has not been shown, the identity of the principal is not necessary if the existence of a guilty principal is proven. Even if the principal is not charged with the crime, a person may be convicted of aiding and abetting the offense. The failure to prosecute or obtain a prior conviction of a principal, such as where he may have been granted immunity or pleaded to a lesser offense, does not preclude conviction of the aider and abettor, as long as the commission of the crime by a principal is proved. Conduct designed to aid another in commission of a crime- a person who engages in conduct designed to aid another to commit a crime that would establish his complicity if the crime were committed by such other person, is guilty of an attempt to commit the crime, although the crime is not committed or attempted by such other person. Success in aiding is irrelevant under MPC.

36

Common law- an accessory could not be convicted without the prior conviction of the principal offender. The principals flight, death, or acquittal bars prosecution of the accessory. And if the principal were pardoned or his conviction reversed on appeal, the accessorys conviction could not stand. In every way and accessory followed like a shadow, his principal. This procedural bar applied only to the prosecution of accessories in felony cases. In misdemeanor cases, where all participants were deemed principals, a prior acquittal of the actual perpetrator did not prevent the subsequent conviction of a person who rendered assistance. In felony cases a principal in the second degree could be convicted notwithstanding the prior acquittal of the first degree principal. Whoever directly commits any act constituting an offense defined in any law of the US, or aids, abets, counsels, commands, induces, or procures its commission, is a principal. Unlike mens rea for attempts, an aider and abettors intent regarding the substantive offense is the same intent required for conviction as a principal. To prove liability as an aider and abettor the government must establish beyond a reasonable doubt that the accused had the specific intent to facilitate the commission of a crime by someone else- and this is an element that need not be established for conviction on the underlying offense. As with conspiracy and attempt, one who abandons the criminal act may not be held criminally liable. The abandonment needs to be voluntary and there must be a complete abandonment. Often it is necessary to communicate the withdrawal from the accessory activity to law enforcement. Pg. 519 MPC Liability for conduct of another; complicity.

US v. Cruz (2nd Cir. 2004) heroin deal at Boston Market doesnt go very well Rule: an aiding and abetting conviction must be premised on more than evidence of a general cognizance of criminal activity or suspicious circumstances. To convict D on a theory of aiding and abetting, the government must prove that the underlying crime was committed by a person other than D and that D acted, or failed to act in a way that the law required him to act, with the specific purpose of bringing about the underlying crime. To prove that d acted with that specific intent, the government must show that he knew of the proposed crime. Although the government may prove the requisite knowledge and specific intent by circumstantial evidence, that evidence must still include some indicia of the specific elements of the underlying crime. Ds mere presence at the scene of the crime or association with wrongdoers does not constitute intentional participation in the crime, even if D had knowledge of the criminal activity. 2 classes of mental states for accomplice liability; (1) whatever mental state the substantive crime requires, including possible attendant circumstances, (2) the purpose to commit the act of aiding the principal in committing the substantive crime and, in many jurisdictions, the purpose to achieve the result required by the crime. MPC seems to require the accomplice to have the purpose of aiding the principal both in committing the act elements of the underlying substantive crime and concerning the accomplices attitude toward the attendant circumstances of that crime. Except for strict liability offenses, every crime has two components- (1) an act or omission, sometimes called the actus reus, and (2) a necessary mental state, sometimes called the mens rea. This principle applies to aiding and abetting liability as well as direct liability. An aider and abettor must do something and have a certain mental state.

37

Week 12 11/1 and 11/3 Defenses, self defense I. Defenses a. Defense counsel can request that the court dismiss the charges because there is insufficient proof of the crime. This is called a motion for a judgment acquittal or directed verdict. b. 5 general categories of Defense: i. Failure of Proof: 1. Because of the defense the prosecution is unable to prove all the required elements of the offense, the objective conduct, circumstances, and result elements and their corresponding culpability requirements. 2. Offense Modifications: a. Modify or refine the criminalization decision embodied in the definition of the particular offense. 3. Justifications: 4. Actor that engages in conduct that is not culpable because its benefits outweigh the harm or evil of the offense 5. Excused: a. An actor admits the harm or evil but nonetheless claims an absence of personal culpability 6. Nonexculpatory Public Policy Defense: a. An actor who admits the harm or evil and his culpability but relies upon an important public policy interest. Self Defense (MPC) a. The use or force upon or toward another person is justifiable when the actor believes that such force is immediately necessary for the purpose of protecting himself against the use of unlawful force by such other person on the present occasion. b. Limitations on Justifying Necessity for Use of Force i. The use of force is NOT justifiable under this section: 1. To resist an arrest actors knows is being made by peace officer 2. To resist force used by the occupier or possessor of property 3. Unless actor believes such force is necessary to protect himself against death or serious bodily injury 4. The actor provoked the use of force against himself c. Definitions (MPC) i. Unlawful Force 1. Force, including confinement that is employed without the consent of the person against whom it is directed of which constitutes an offense, not amounting to privilege to use such force. 2. Deadly Force a. Force that the actor uses with the purpose of causing or that he knows to create a substantial risk of causing death or serious bodily injury. Self Defense (Common Law) a. The defendant is threatened with the use, or threat of imminent use of b. Unlawful
38

II.

III.

c. Force d. That does or could cause physical injury and e. Which a reasonable person would believe could not be avoided without the use of physical force and the actor f. Defends by using only reasonable amount of force, and g. Was not responsible for the situation that promoted the need to use such force. ** 3 General Principles for using Self-defense:** (a) Necessity, (b) Proportionality (c) Reasonable Belief There are two types of imperfect self-defense: (1) In some courts, a non-deadly aggressor who meets a deadly response is required to retreat before resorting to deadly force. If he fails to retreat, his self-defense is imperfect. (2) In many states, one who kills another because he unreasonably believes the circumstances justify the killing (unreasonably believes that the force she was defending against was deadly, or was imminent, or was unlawful, or was necessary), the self-defense is imperfect. In either case, the imperfect self-defense is insufficient to justify the crime, but does negate malice. So the homicide is bumped down from murder to voluntary manslaughter. The first type is a pretty limited exception. It applies only in those states that require a nondeadly aggressor to retreat when facing a deadly response, and only when that nondeadly aggressor fails to retreat as required.

Beatty v. Commonwealth (Ky. 2003) every kind of drug and meth lab in 1 car Rule: the right of an accused in a criminal trial to due process is, in essence, the right to a fair opportunity to defend against the states accusations. An exclusion of evidence will almost invariably be declared unconstitutional when it significantly undermines fundamental elements of Ds defense. D has the right to introduce evidence that another person committed the offense with which he is charged. A trial court may only infringe upon this right when the defense theory is unsupported, speculative, and farfetched and could thereby confuse or mislead the jury. Evidence is not automatically admissible simply because it tends to show that someone else committed the offense. In the same way, evidence of opportunity alone is sufficient to guarantee admissibility. Simply showing that the alleged alternative perpetrator, however is in a different position. Appellate courts have almost invariably reversed when proffered evidence of both motive and opportunity has been excluded by the trial court.

State v. Deffebaugh (Kan. 2004) cocaine deal with marked bills Rule: notice must be given before evidence of alibi is given, in Kansas. Purpose is to protect state from easily fabricated defenses. An alibi places D at the relevant time in a different place than the scene involved and so removed therefrom as to render it impossible for the accused to be the guilty party. By relying on this statute an inference can be drawn that the testimony would prove that D was not at the scene even though the testimony was inadmissible. Notice is required for alibi defense, not eyewitness testimony.

39

People v. Romero (Cal. App. 1999) street fighting is not culturally excusable Rule: An actual but unreasonable fear would negate malice aforethought. Did D actually believed he was in imminent danger of death or great bodily injury; and whether such a belief was objectively reasonable.

Common law self defense- D must justify conduct that would otherwise be criminal: (1) D is threatened with the use, or threat of the imminent use, of (2) unlawful (3) force (4) that does or could cause physical injury and (5) which a reasonable person would believe could not be avoided without the use of physical force, and the actor (6) defends by using only a reasonable amount of force and (7) was not responsible for the situation that prompted the need to use such force. Note that self-defense provides a complete defense to the charge, which means D is found not guilty if the elements are established. In some cases the same evidence may be used to satisfy more than one element of the defense. The reasonableness determination will often depend on the amount of force threatened.

People v. Goetz (NY 1986) little shit shoots 4 punks on the subway Rule: use of force is permitted under certain circumstances. A person may use physical force upon another person when and to the extent he reasonably believes such to be necessary to defend himself or a 3rd person from what he reasonably believes to be the use or imminent use of unlawful physical force by such other person. A person may not use deadly physical force upon another person under circumstances specified unless (a) he reasonably believes that such other person is using or about use deadly physical force or (b) he reasonably believes that such other person is committing or attempting to commit a kidnapping, forcible rape, forcible sodomy or robbery. MPC self-defense- Any culpability which arises from a mistaken belief in the need to use such force should be no greater than the culpability such a mistake would give rise to if it were made with respect to an element of a crime. A D charged with murder (or attempted murder) need only show that he believed that the use of deadly force was necessary to protect himself against death, serious bodily injury, kidnapping or forcible sexual intercourse to prevail on a self-defense claim. If Ds belief was wrong, and was recklessly, or negligently formed, however, he may be convicted of the type of homicide charge requiring only a reckless or negligent, as the case may be, criminal intent. General principles for using self-defense are: (a) necessity, (b) proportionality, and (c) reasonable belief. If the threat to the individual is not immediate then the use of force is usually not considered necessary. D, asserting self-defense must actually hold the belief (subjective), and it must be a reasonable belief of an imminent and unlawful harm (objective). The status of the accused as an aggressor precludes a claim of self-defense. The common law rule that selfdefense must be proven by D under the preponderance standard. Almost every state has abandoned that approach, and now require D to meet the initial burden of producing evidence to establish self-evidence, and then the prosecution must demonstrate the lack of self-defense beyond a reasonable doubt for the jury to convict. A minority of the states require D to retreat to a place of safety, if one is reasonably available, before using deadly force. The retreat rule does not require a person to place himself or herself in any danger, and the avenue
40

of escape must be reasonably available to the person. If such a means to avoid the confrontation is available, the self-defense may not be offered and the person can be held liable for the harm to the victim.

State v. Sandoval (Or. 2007) D gets rear ended and kills the guy Rule: no requirement to retreat before using deadly force to defend against the imminent use of deadly physical force by another. A person is justified in using physical force upon another person to defend himself from what he reasonably believes to be the use or imminent use of unlawful physical force. If defending, a person may only use that degree of force which he reasonably believes to be necessary. The burden of proof is on the state to prove beyond a reasonable doubt that D did not act in self-defense. There are certain limitations on the use of deadly physical force. D is not justified in using deadly physical force against another person in self-defense unless he reasonably believed that the other person was using or about to use unlawful deadly physical force against him and/or committing or attempting to commit a felony involving the use or threatened imminent use of physical force against a person. Even in the situation where one of these threatening circumstances is present, the use of deadly physical force is justified only if it does not exceed the degree of force which D reasonably believes to be necessary in the circumstances. The danger justifying the use of deadly force must be absolute, imminent, and unavoidable, and a necessity of taking human life must be actual, present, urgent and absolutely or apparently absolutely necessary. There must be no reasonable opportunity to escape to avoid the affray and there must be no other means of avoiding or declining the combat.

The Castle Doctrine- even in those jurisdictions that require retreat before using deadly force, a person need not withdraw in their own home, or in the area immediately adjoining it- called the curtilage. Regardless of any general theory to retreat as far as practicable before one can justify turning upon his assailant and taking life in self-defense, the law imposes no duty to retreat upon one who, free from fault in bringing on a difficulty, is attacked at or in his or her own dwelling or home. Upon the theory that a mans house is his castle, and that he has a right to protect it and those within it from intrusion or attack, the rule is practically universal that when a person is attacked in his own dwelling he may stand at bay and turn on and kill his assailant if this is apparently necessary to save his own life or to protect himself from great bodily harm. Common law retreat rule does not apply when an intruder attempts to invade a home. MPC Self-Defense pg. 591.

Week 13 11/8 and 11/10 Self defense, defense of others and property, necessity and duress, entrapment I. Defense to Others a. Elements i. Burden of Proof 1. A defendant can be required to prove the affirmative defenses that he asserts.
41

ii. Amount of Force 1. A person may use only that force which is necessary in view of the nature of the attack; any use of excessive force is not justified and a homicide which results therefrom is unlawful. iii. Reasonable Belief That Intervention Was Lawful 1. Shifts the emphasis to the defendants reliance on reasonable appearances rather than exposing him to the peril of liability for defending another where appearances were deceiving and there was no actual imminent danger. iv. Level of Danger 1. An intervenor cannot act until the party whom the intervenor is defending is immediately threatened. b. Elements (MPC) i. The use of force upon or toward the person of another is justifiable to protect a third person when; 1. The actor would be justified in using such force to protect himself against the injury he believes to be threatened to the person who he seeks to protect; and 2. The actor believes that his intervention is necessary for the protection of such other person. ii. An actor is obliged to retreat, surrender the possession of a thing or to comply with a demand before using force in self-protection, he is not obliged to do so before using force for the protection of another person, unless he knows that he can thereby secure the complete safety of such other person. Self defense- D is not the aggressor, he reasonably believes deadly force is necessary to repel imminent threat of unlawful deadly force or deadly force is necessary because it is likely or intended to cause death or serious bodily harm. Aggressor- status can shift when non-deadly aggressor is responded to with deadly aggression- retreat may be necessary before status shifts (minority rule). Majority rule- doesnt have to retreat unless D shares some fault for the affray (mutual combat or was nondeadly aggressor). Minority rule- must attempt retreat 1st but only if it is safe to do so and not in his own home. If belief is unreasonably- no self defense. But imperfect self defense may apply (voluntary manslaughter). Fault in starting altercation is reckless. Common law self defense- belief objectively reasonable. Threat/harm must be imminent. No need to retreat. MPC self defense- if subjective belief is reckless or negligent, may be still be convicted of the negligent crime. Need to take defensive action except at home. Most states do not put burden of proof on D for self defense. Only need to bring some evidence. Defense of others- affirmative defense, requires proportionality, reasonable belief, requires 3rd party be in imminent danger of serious bodily harm or death.

42

Alter ego rule- if 3rd party would have had right of self defense, intervener can assert this defense (traditional rule). Apparent right of self defense- modern/majority rule. Non-deadly force is justified if reasonably believes it is necessary to prevent imminent, unlawful dispossession of property. Deadly force is not justified to avoid being dispossessed of personal property.

Bonner v. State (Ala. 1998) battered women syndrome Rule: expert testimony regarding battered woman syndrome can be admitted to help the jury not only to understand the battered woman syndrome but also determine whether D has reasonable grounds for an honest belief that she was in imminent danger when considering the issue of self-defense.

State v. Cook (WV 1999) wife defends husband a bit too well Rule: a person is justified in using force to protect a 3rd party from unlawful force by an aggressor. The right of self defense of another usually falls under the rubric of self defense. One simply steps into the shoes of the victim and is able to do only as much as the victim himself would lawfully be permitted to do. Common law self defense of others- doctrine was imposed only as a defense when a homicide occurred in defense of a member of ones family. The privilege of using deadly force did not include authority for interveners to protected 3rd persons who were strangers to the intervener. Alter ego rule- D using deadly force to defend a person who was not entitled to use deadly force would be held criminally liable. Reasonable belief rule- an intervener who acts in defense of another is not liable if his or her actions were reasonable under the circumstances. D may be legally justified in killing to defend another, even if the intervener acted under a mistaken belief as to who was at fault, provided his belief was reasonable. D must show by sufficient evidence that he used reasonable force, including deadly force, in a situation where D had a reasonable belief of the lawfulness of his or her intervention on behalf of another person who was in imminent danger of death or serious bodily harm from which such person could save himself only by using force, including deadly force, against his or her assailant, but was unable to do so. Proportionality rule- a person may use only that force which is necessary in view of the nature of the attack; any use of excessive force is not justified and a homicide which results there from is unlawful. The actor must actually believe that another is in danger and that belief must be a reasonable one. An intervener cannot act until the party whom the intervener is defending is immediately threatened. The reasonableness of Ds belief and actions in using deadly force must be judged in the light of the circumstance in which he acted at the time and is not measured by subsequently developed facts. Reasonable belief standard is composed of an objective element, a reasonable belief that force is necessary, and a subjective element, an actual belief that force is necessary. A person may use only that force which is necessary in view of the nature of the attack; any use of excessive force is not justified and a homicide which results there from is unlawful.
43

MPC defense of others pg 609 MPC duress pg 643 MPC choice of evils pg 631 MPC defense of property pg 618 MPC entrapment pg 661

Gaitlin v. US (DC App. 2003) principal and employees are nuts! Rule: one is justified in using reasonable force to protect his or her property from trespass or theft, when he or she reasonably believes that his or her property is in immediate danger of such an unlawful interference and that the use of such force is necessary to avoid that danger. In the absence of excessive or unnecessary force by an arresting officer, a person may not use force to resist an arrest by one he knows or has good reason to believe is an authorized police officer, engaged in the performance of his duties, regardless of whether the arrest was unlawful in the circumstances. D is not required to prove that he acted in defense of his property. Rather, if evidence of defense of property is present, the burden is on the government to prove beyond a reasonable doubt that D did not act in defense of his property. A person may use as much force as is reasonably necessary to eject a trespasser from his or her property, and that if he or she uses more force than is necessary, he or she is guilty of assault.

People v. Fontes (Colo. App. 2004) bum forges big check to feed kids Rule: choice of evils is a statutory defense applicable when the alleged crimes were necessary as an emergency measure to avoid an imminent public or private injury that is about to occur by reason of a situation occasioned or developed through no conduct of the actor and which is of sufficient gravity to outweigh the criminal conduct. D must offer proof of sudden and unforeseen emergence of a situation requiring his or her immediate action to prevent an imminent injury. An affirmative defense such as choice of evils provides a legal justification for otherwise criminally culpable behavior. D who asserts an affirmative defense admits the doing of a charged act, but seeks to justify the act on grounds deemed by law to be sufficient to avoid criminal responsibility. A D who has a reasonable legal alternative as a means to avoid the threatened injury is foreclosed from asserting a choice of evils defense. A D who seeks to assert a choice of evils defense must offer evidence that the criminal conduct at issue did not exceed that reasonably necessary to avoid the impending injury. Before instructing the jury on the choice of evils defense, the trail court must look at the evidence in the light most favorable to D and determine whether the facts could justify the crimes charged. On appeal, we must determine as a matter of law whether Ds offer of proof, considered in the light most favorable to him, is substantial and sufficient to support the statutory defense.

44

Geljack v. State (Ind. App. 1996) driving on suspended license Rule: emergency defense mitigates culpability. There is a difference between affirmative defenses that establish separate and distinct facts in mitigation of culpability and affirmative defenses that negate an element of the crime. It is only unconstitutional to place the burden of persuasion for an affirmative defense on D when proving the defense becomes tantamount to requiring D to negate an element of the crime.

US v. Maxwell (1st Cir. 2001) idiot trespassed onto live firing range to protest drills Rule: necessity defense requires D to show that he (1) was faced with a choice of evils and chose the lesser evil, (2) acted to prevent imminent harm, (3) reasonably anticipated a direct causal relationship between his acts and the harm to be averted, and (4) has not legal alternative but to violate the law. D must demonstrate cause and effect between an act of protest and the achievement of the goal of the protest by competent evidence. He cannot will a causal relationship into being simply by the fervor of his convictions no matter how sincerely held.

Jacobson v. US (1992) guys buys porn and gets the wrong shit! Rule: government agents may not originate a criminal design, implant in an innocents persons mind the disposition to commit a criminal act, and then induce commission of the crime so that the government may prosecute. Where the government has induced an individual to break the law and the defense of entrapment is at issue, the prosecution must prove beyond a reasonable doubt that D was dispose to commit the criminal act prior to first being approached by government agents.

45

Вам также может понравиться